Provable Question Stems Flashcards

1
Q

-Which one of the following most accurately expresses the main conclusion of the argument?

A

Conclusion Question Stem Example

How well did you know this?
1
Not at all
2
3
4
5
Perfectly
2
Q

-If the statement above is true, which one of the following must also be true?
-Which one of the following follows logically from the statements above?

A

Inference Question Stem Example

How well did you know this?
1
Not at all
2
3
4
5
Perfectly
3
Q

-Which one of the following is most strongly supported by the information above?
-The statements above, if true, support which one of the following?

A

Most Strongly Supported Question Stem Example

How well did you know this?
1
Not at all
2
3
4
5
Perfectly
4
Q

-Which one of the following most logically completes the argument?
-The conclusion of the argument is strong supported if which one of the following completes the argument?

A

Fill In Question Stem Example

How well did you know this?
1
Not at all
2
3
4
5
Perfectly
5
Q

-Max and John disagree over whether:
-The statements above provide the most support for holding that Max would disagree with John about what statement?

A

Controversy Question Stem Example

How well did you know this?
1
Not at all
2
3
4
5
Perfectly
6
Q

-The argument assumes which of the following?
-Which one of the following is an assumption required by the argument?

A

Necessary Assumption Question Stem Example

How well did you know this?
1
Not at all
2
3
4
5
Perfectly
7
Q

-Which one of the following most accurately describes how the argument proceeds?

A

Method Question Stem Example

How well did you know this?
1
Not at all
2
3
4
5
Perfectly
8
Q

-The reference plays which of the following roles in the argument?
-The statement serves which of the following functions?

A

Argument Part Question Stem Example

How well did you know this?
1
Not at all
2
3
4
5
Perfectly
9
Q

-The reasoning in the argument is most vulnerable to criticism on the grounds that the argument:
-Which most accurately describes a flaw in the reasoning of the argument?

A

Classic Flaw Question Stem Example

How well did you know this?
1
Not at all
2
3
4
5
Perfectly
10
Q

-The reasoning is flawed because it presumes without giving sufficient justification that:
-The advertisements reasoning is most vulnerable to criticism on the grounds that:

A

Loophole Flaw Question Stem Example

How well did you know this?
1
Not at all
2
3
4
5
Perfectly
11
Q

-Which one of the following conforms most closely to the principle illustrated above?
-Which one of the following propositions is best illustrated by the situation described?

A

Principle Conform Question Stem Example

How well did you know this?
1
Not at all
2
3
4
5
Perfectly
12
Q

-Which one of the following exhibits flawed reasoning that is most parallel to the argument above?

A

Parallel Flaw Question Stem Example

How well did you know this?
1
Not at all
2
3
4
5
Perfectly